Convergence of a recursively defined sequence

Using a lower bound of 0 instead would not let us make those three steps, but maybe we could make up some other way to prove decreasingness. It might not be as elegant though.In summary, the author chooses ##\sqrt{5}## to be the lower bound for the sequence because it is a tighter limit that is needed to prove that the sequence is decreasing. While choosing 0 as the lower bound would satisfy the requirement of being bounded below, it may not be sufficient to prove decreasingness. The upper bound of 5 is not necessary for proving convergence, but it is helpful in showing that the actual limit is ##\sqrt 5##.
  • #1
Mr Davis 97
1,462
44
I have the following sequence: ##s_1 = 5## and ##\displaystyle s_n = \frac{s_{n-1}^2+5}{2 s_{n-1}}##. To prove that the sequence converges, my textbook proves that the following is true all ##n##: ##\sqrt{5} < s_{n+1} < s_n \le 5##. I know to prove that this recursively defined sequence converges, we have to show that it is decreasing and that it is bounded below. As such, I have a few questions: why does the author choose ##\sqrt{5}## to be the potential lower bound? Would it be just as valid to prove use 0 as the number we could prove to be a lower bound? Also, why is that 5 there? Is it at all necessary to the proof?
 
Physics news on Phys.org
  • #2
Choosing 0 would satisfy the 'bounded below' bit, but that's only one of the two requirements. I suspect that those tighter limits of 5 and ##\sqrt 5## will be needed in proving the other requirement - that the series is decreasing.
 
  • #3
andrewkirk said:
Choosing 0 would satisfy the 'bounded below' bit, but that's only one of the two requirements. I suspect that those tighter limits of 5 and ##\sqrt 5## will be needed in proving the other requirement - that the series is decreasing.
But, just say that I want to prove that the sequence is decreasing. How would I know beforehand that I must prove that ##s_n > \sqrt{5}## for all ##n##? It seems to me that it is not immediately obvious that I must prove that ##s_n > \sqrt{5}##. Where does the ##\sqrt{5}## come from?
 
  • #4
Mr Davis 97 said:
But, just say that I want to prove that the sequence is decreasing. How would I know beforehand that I must prove that ##s_n > \sqrt{5}## for all ##n##?
You won't and it is not necessary. Any finite lower bound as e.g. zero will do to prove convergence. If you want to prove what the limit is, you need a proposal. I haven't checked, but maybe ##\sqrt{5}## comes into play doing that.
It seems to me that it is not immediately obvious that I must prove that ##s_n > \sqrt{5}##. Where does the ##\sqrt{5}## come from?
Probably from ##s=\dfrac{s^2+5}{2s}\,.## If ##s_{n+1} < s_n## and ##s_0=5##, then the limit must be around ##s## and it can only approach from above.
 
  • #5
fresh_42 said:
You won't and it is not necessary. Any finite lower bound as e.g. zero will do to prove convergence. If you want to prove what the limit is, you need a proposal. I haven't checked, but maybe ##\sqrt{5}## comes into play doing that.

Probably from ##s=\dfrac{s^2+5}{2s}\,.## If ##s_{n+1} < s_n## and ##s_0=5##, then the limit must be around ##s## and it can only approach from above.
So instead of proving that ##\sqrt{5} < s_{n+1} < s_n \le 5## is true for all ##n##, could I just as well prove that ##0 < s_{n+1} < s_n \le 5## to prove convergence?

If this is the case, why is it that author just goes ahead and uses ##\sqrt{5}##?
 
  • #6
Mr Davis 97 said:
So instead of proving that ##\sqrt{5} < s_{n+1} < s_n \le 5## is true for all ##n##, could I just as well prove that ##0 < s_{n+1} < s_n \le 5## to prove convergence?
For convergence, yes.
If this is the case, why is it that author just goes ahead and uses ##\sqrt{5}##?
Because this is the limit.

It's like saying: "I'm less than 20 ft tall." You would probably use a number which is closer to the truth, even if you only want to say that you're not infinitely tall.
 
  • Like
Likes Mr Davis 97
  • #7
fresh_42 said:
For convergence, yes.

Because this is the limit.

It's like saying: "I'm less than 20 ft tall." You would probably use a number which is closer to the truth, even if you only want to say that you're not infinitely tall.
Just to clarify though, since I don't technically know that ##\sqrt{5}## is the limit beforehand, in practice would I choose something more obvious as a lower bound, like ##0##?
 
  • #8
Mr Davis 97 said:
Just to clarify though, since I don't technically know that ##\sqrt{5}## is the limit beforehand, in practice would I choose something more obvious as a lower bound, like ##0##?
I would do so. The crucial part is ##s_{n+1} < s_n##. The rest of a convergence proof is more or less obvious, because of ##(s_n)\subseteq [0,5]##.
 
  • #9
fresh_42 said:
I would do so. The crucial part is ##s_{n+1} < s_n##. The rest of a convergence proof is more or less obvious, because of ##(s_n)\subseteq [0,5]##.
Is the fact that 5 is an upper bound at all necessary to show convergence? Don't we just have to know that the sequence is decreasing and that there is a lower bound?
 
  • #10
Mr Davis 97 said:
Is the fact that 5 is an upper bound at all necessary to show convergence? Don't we just have to know that the sequence is decreasing and that there is a lower bound?
With monotony the first sequence element is automatically an upper bound, so necessary is a bit difficult to say, as it is as obvious. Personally, I like ##(s_n) \subseteq [0,5]## because one sees at once, that it is a compact set and thus the limits are within. This way it isn't necessary to think about the right end. It's more a matter of convenience to have a compact set than a necessity.

If you apply pure logic, then things are more complicated anyway. E.g. you need an argument why ##s_n < 5 ## and ##s_{n+1}< s_n## implies ##s_{n+1} < 5## (transitivity of the ordering plus induction, resp. Peano on the index set).
 
  • Like
Likes Mr Davis 97
  • #11
Mr Davis 97 said:
Is the fact that 5 is an upper bound at all necessary to show convergence? Don't we just have to know that the sequence is decreasing and that there is a lower bound?
The upper bound of 5 is not used anywhere in the proof of convergence. It is not even used in proving that the actual limit is ##\sqrt 5##.

The lower bound of ##\sqrt 5## is a key part of proving the sequence is decreasing, which is a necessary part of proving convergence. The proof of decreasingness is easy given that lower bound - three short steps. Using a lower bound of 0 instead would not work.
 
Last edited:
  • #12
fresh_42 said:
With monotony the first sequence element is automatically an upper bound, so necessary is a bit difficult to say, as it is as obvious. Personally, I like ##(s_n) \subseteq [0,5]## because one sees at once, that it is a compact set and thus the limits are within. This way it isn't necessary to think about the right end. It's more a matter of convenience to have a compact set than a necessity.

If you apply pure logic, then things are more complicated anyway. E.g. you need an argument why ##s_n < 5 ## and ##s_{n+1}< s_n## implies ##s_{n+1} < 5## (transitivity of the ordering plus induction, resp. Peano on the index set).

Makes sense. Last thing. What would be the best way to prove that ##s_{n+1} < s_n## in this case? Should I start on the LHS and make an argument that looks something like ##\displaystyle s_{n+1} = \frac{s_{n}^2+5}{2 s_{n}} < \cdots < s_n##, OR should I make an argument that starts with ##s_{n+1} < s_n \implies \frac{s_{n}^2+5}{2 s_{n}} < s_n \implies s_n > \sqrt{5}## and prove that ##s_n > \sqrt{5}## for all ##n##?
 
  • #13
Mr Davis 97 said:
Makes sense. Last thing. What would be the best way to prove that ##s_{n+1} < s_n## in this case? Should I start on the LHS and make an argument that looks something like ##\displaystyle s_{n+1} = \frac{s_{n}^2+5}{2 s_{n}} < \cdots < s_n##, OR should I make an argument that starts with ##s_{n+1} < s_n \implies \frac{s_{n}^2+5}{2 s_{n}} < s_n \implies s_n > \sqrt{5}## and prove that ##s_n > \sqrt{5}## for all ##n##?
It's equivalent - because we have only positive numbers - to ##\sqrt{5} < s_n## and that also answers your question, why the square root of five as lower bound comes into play. I would do it by induction. Whether it is the best way is a matter of taste. There's probably a more elegant way using calculus.
 
  • #14
Mr Davis 97 said:
OR should I make an argument that starts with ##s_{n+1} < s_n \implies \frac{s_{n}^2+5}{2 s_{n}} < s_n \implies s_n > \sqrt{5}## and prove that ##s_n > \sqrt{5}## for all ##n##?
That would not lead anywhere. It is called 'affirming the consequent'. If ##A\to B## then proving ##B## tells us nothing about the truth or otherwise of ##A##. In any case, that ##s_n>\sqrt 5## has already been proved by the text.

Look at the formula ##s_{n+1}=\frac{s_n{}^2+5}{2s_n}##. If it wasn't for that 5 in the numerator we could cancel out the ##s_n## in the denominator and get a simpler expression that might give us a useful inequality. Using one of the inequalities proved in the text, can we replace the 5 by something greater than or equal to 5 that allows us to cancel out the ##s_n## in the denominator?
 
  • #15
andrewkirk said:
That would not lead anywhere. It is called 'affirming the consequent'. If ##A\to B## then proving ##B## tells us nothing about the truth or otherwise of ##A##. In any case, that ##s_n>\sqrt 5## has already been proved by the text.

Look at the formula ##s_{n+1}=\frac{s_n{}^2+5}{2s_n}##. If it wasn't for that 5 in the numerator we could cancel out the ##s_n## in the denominator and get a simpler expression that might give us a useful inequality. Using one of the inequalities proved in the text, can we replace the 5 by something greater than or equal to 5 that allows us to cancel out the ##s_n## in the denominator?
If we know that ##s_n^2 >5##, then ##2 s_n^2 > s^2_n + 5##, so ##s_{n+1} = \frac{s_n^2+5}{2s_n} < \frac{2s_n^2}{2s_n} = s_n##. I guess that makes sense... My question then is if I were to approach this problem on my own without the book, how would I know beforehand that ##s^2_n > 5##?
 
  • #16
Mr Davis 97 said:
My question then is if I were to approach this problem on my own without the book, how would I know beforehand that ##s^2_n > 5##?
You would a) assume the sequence converges, b) therefore conclude that ##s_{n+1}-s_n## is narrowing down, so for big enough ##n## we c) get ##s=\dfrac{s^2+5}{2s}## with only a small error. But this means d) ##s=\sqrt{5}## which is e) our candidate for the limit. As we start with ##5## and ##\sqrt{5}<5## there must f) be a decrease somewhere. On the other hand, we g) don't have any hints that the sequence jumps back and forth, so h) ##s_{n+1} < s_n## is a natural assumption to be checked.
 
  • #17
Typically with a sequence, one calculates the first few values to look for patterns. After three steps the number is very close to ##\sqrt 5## and the presence of the numeral 5 in the formula suggests trying a simple function of 5, such as ##\sqrt 5## as the limit. We also notice that the sequence is decreasing in those first three steps, which suggests a downwards asymptotic approach.

By the way, once we know the sequence is decreasing, we can directly prove the sequence has a limit of ##\sqrt 5## as follows. The reduction from one sequence element to the next is
\begin{align*}
s_n-s_{n+1}
&=
s_n-\frac{s_n{}^2 + 5}{2s_n}\\
&=
\frac{s_n{}^2 - 5}{2s_n}\\
&=
\frac{(s_n - \sqrt 5)(s_n + \sqrt 5)}{2s_n}\\
&=
(s_n - \sqrt 5)\left(\frac12 +\frac{\sqrt 5}{2s_n}\right)\\
&>
\frac12 (s_n - \sqrt 5)
\end{align*}
From which we observe that:
\begin{align*}
s_{n+1}-\sqrt 5
&= (s_n-\sqrt 5) - (a - b)\\
&<
(s_n-\sqrt 5) -\frac12 (s_n-\sqrt 5)\\
&=
\frac12 (s_n-\sqrt 5)
\end{align*}
So the distance of ##s_n## from ##\sqrt 5## is more than halving at every step, so it must go to that limit.
 
  • #18
As far as I can see, the formula is the recursive formula for solving the square root of 5...
 
  • #19
Svein said:
As far as I can see, the formula is the recursive formula for solving the square root of 5...
Yes, looks like Newton's method to take roots by hand.
 

1. What is convergence of a recursively defined sequence?

Convergence of a recursively defined sequence refers to the behavior of a sequence where each term is defined in terms of the previous terms. In other words, the value of each term depends on the value of the previous term. The sequence is said to converge if the terms get closer and closer to a single value as the sequence progresses.

2. How do we determine if a recursively defined sequence converges?

To determine if a recursively defined sequence converges, we can use the limit test. This involves taking the limit of the sequence as the number of terms approaches infinity. If the limit exists and is a finite number, then the sequence converges. If the limit does not exist or is infinite, then the sequence does not converge.

3. Can a recursively defined sequence converge to more than one value?

No, a recursively defined sequence can only converge to one value. This is because the recursive definition of the sequence means that each term is dependent on the previous term, so as the sequence approaches a single value, all subsequent terms will also approach that same value.

4. What is the difference between convergence and divergence of a recursively defined sequence?

Convergence and divergence are two opposite behaviors of a recursively defined sequence. As mentioned earlier, a sequence converges if the terms get closer and closer to a single value as the sequence progresses. On the other hand, a sequence diverges if the terms do not approach a single value and instead keep increasing or decreasing without bound.

5. Can a recursively defined sequence converge if the limit of its terms does not exist?

No, a recursively defined sequence cannot converge if the limit of its terms does not exist. Convergence requires the limit of the terms to exist and be a finite number. If the limit does not exist, then the sequence either diverges or oscillates between different values and does not approach a single value.

Similar threads

Replies
11
Views
1K
  • Calculus
Replies
1
Views
73
  • Calculus and Beyond Homework Help
Replies
17
Views
1K
Replies
15
Views
2K
  • Calculus
Replies
7
Views
2K
Replies
1
Views
1K
Replies
9
Views
874
Replies
1
Views
1K
Back
Top